Finding W perp of inner product space

Click For Summary
SUMMARY

The discussion focuses on finding the orthogonal complement W-perp of the subspace W in the polynomial space P5(R), defined by the conditions p(0) = 0, p'(0) = 0, and p''(0) = 0. The participants identify that the polynomial p(x) can be expressed as p(x) = ax^5 + bx^4 + cx^3, leading to the conclusion that the basis for W is {x^5, x^4, x^3}. To find W-perp, one must ensure that any polynomial q(x) is orthogonal to each vector in this basis through the inner product defined as = ∫p(x)q(x) dx.

PREREQUISITES
  • Understanding of polynomial spaces, specifically P5(R).
  • Knowledge of inner product definitions and properties.
  • Familiarity with solving systems of equations in matrix form.
  • Ability to manipulate and integrate polynomials.
NEXT STEPS
  • Learn how to compute orthogonal complements in inner product spaces.
  • Study the properties of polynomial bases and their implications in functional analysis.
  • Explore techniques for integrating polynomials, including substitution methods.
  • Investigate the application of linear algebra in solving polynomial equations.
USEFUL FOR

Mathematicians, students studying linear algebra or functional analysis, and anyone interested in the properties of polynomial spaces and inner product spaces.

trap101
Messages
339
Reaction score
0
Consider P5(R) together with innner product < p ,q > = ∫p(x)q(x) dx. Find W-perp if W = {p(x) \in P5(R) : p(0) = p'(0) = p''(0) = 0}

Attempt: I am having trouble with the condition. I always have trouble with these conditions. SO as of now I am going to let q(x) be the standard basis of P5(R). Now I don't know how to apply the condition to p(x).

After I do apply the condition I would take the inner product and have it set equal to 0. I should have a set of equations that I can solve in matrix form. This should produce some free variables from where I can obtain vectors for W-perp. So the concept is understood...I just can't seem to use the conditions appropriately...

thanks.
 
Physics news on Phys.org
Write p(x) out as a polynomial. Your conditions are giving you information about the coefficients of the polynomial. What does p(0)=0 tell you?
 
trap101 said:
Consider P5(R) together with innner product < p ,q > = ∫p(x)q(x) dx. Find W-perp if W = {p(x) \in P5(R) : p(0) = 0,p'(0) =0, p''(0) = 0}

Attempt: I am having trouble with the condition. I always have trouble with these conditions. SO as of now I am going to let q(x) be the standard basis of P5(R). Now I don't know how to apply the condition to p(x).

After I do apply the condition I would take the inner product and have it set equal to 0. I should have a set of equations that I can solve in matrix form. This should produce some free variables from where I can obtain vectors for W-perp. So the concept is understood...I just can't seem to use the conditions appropriately...

thanks.

If ##p(x) = a + bx + cx^2 + dx^3 + ex^4 +fx^5##, set ##p(0)=0,p'(0)=0,p''(0)=0##. What does that tell you?
 
LCKurtz said:
If ##p(x) = a + bx + cx^2 + dx^3 + ex^4 +fx^5##, set ##p(0)=0,p'(0)=0,p''(0)=0##. What does that tell you?



Well I set it up the other way so my "a" was with ax5. Well after doing that it turns out that d=e=f= 0 in order for the polynomial to satisfy the condition.

Doing that it turns into p(x) = ax5+bx4+cx3

now I have to multiply this by q(x) and then integrate. Is there any substitution I could possibly do or am I just going to have to literally multiply it out and integrate each component...if that's the case wouldn't that just be mean?
 
trap101 said:
Well I set it up the other way so my "a" was with ax5. Well after doing that it turns out that d=e=f= 0 in order for the polynomial to satisfy the condition.

Doing that it turns into p(x) = ax5+bx4+cx3

now I have to multiply this by q(x) and then integrate. Is there any substitution I could possibly do or am I just going to have to literally multiply it out and integrate each component...if that's the case wouldn't that just be mean?

It might seem a little more manageable if you notice a basis for W is {x^5,x^4,x^3}, so q(x) is in W-perp if it's perpendicular to each vector in the basis.
 
Question: A clock's minute hand has length 4 and its hour hand has length 3. What is the distance between the tips at the moment when it is increasing most rapidly?(Putnam Exam Question) Answer: Making assumption that both the hands moves at constant angular velocities, the answer is ## \sqrt{7} .## But don't you think this assumption is somewhat doubtful and wrong?

Similar threads

  • · Replies 5 ·
Replies
5
Views
4K
Replies
5
Views
2K
  • · Replies 6 ·
Replies
6
Views
2K
Replies
4
Views
3K
  • · Replies 9 ·
Replies
9
Views
2K
  • · Replies 6 ·
Replies
6
Views
2K
  • · Replies 1 ·
Replies
1
Views
2K
  • · Replies 7 ·
Replies
7
Views
4K
  • · Replies 1 ·
Replies
1
Views
1K
  • · Replies 24 ·
Replies
24
Views
4K